2022 AMC 12A Problems/Problem 23

Revision as of 21:19, 11 November 2022 by Stevenyiweichen (talk | contribs) (Created page with "==Problem== Let <math>h_n</math> and <math>k_n</math> be the unique relatively prime positive integers such that \[ \frac{1}{1} + \frac{1}{2} + \frac{1}{3} + \cdots + \frac{...")
(diff) ← Older revision | Latest revision (diff) | Newer revision → (diff)

Problem

Let $h_n$ and $k_n$ be the unique relatively prime positive integers such that

\[ \frac{1}{1} + \frac{1}{2} + \frac{1}{3} + \cdots + \frac{1}{n} = \frac{h_n}{k_n} . \]

Let $L_n$ denote the least common multiple of the numbers $1,2,3,\cdots,n$. For how many integers $n$ with $1 \leq n \leq 22$ is $k_n < L_n$?

Solution

We will use the following lemma to solve this problem.

\begin{lemma} Denote by $p_1^{\alpha_1} p_2^{\alpha_2} \cdots p_m^{\alpha_m}$ the prime factorization of $L_n$. For any $i \in \left\{ 1, 2, \cdots, m \right\}$, denote $\sum_{j = 1}^{\left\lfloor \frac{n}{p_i^{\alpha_i}} \right\rfloor} \frac{1}{j} = \frac{a_i}{b_i}$, where $a_i$ and $b_i$ are relatively prime. Then $k_n = L_n$ if and only if for any $i \in \left\{ 1, 2, \cdots, m \right\}$, $a_i$ is not a multiple of $p_i$. \end{lemma}

Now, we use the result above to solve this problem.

Following from this lemma, the list of $n$ with $1 \leq n \leq 22$ and $k_n < L_n$ is \[ 6, 7, 8, 18, 19, 20, 21, 22 . \]

Therefore, the answer is \boxed{\textbf{(D) 8}}.